2016 AMC 10A Problems/Problem 4

Revision as of 19:29, 3 February 2016 by Benq (talk | contribs)

Problem

The remainder can be defined for all real numbers $x$ and $y$ with $y \neq 0$ by \[\text{rem} (x ,y)=x-y\left \lfloor \frac{x}{y} \right \rfloor\]where $\left \lfloor \tfrac{x}{y} \right \rfloor$ denotes the greatest integer less than or equal to $\tfrac{x}{y}$. What is the value of $\text{rem} (\tfrac{3}{8}, -\tfrac{2}{5} )$?

$\textbf{(A) } -\frac{3}{8} \qquad \textbf{(B) } -\frac{1}{40} \qquad \textbf{(C) } 0 \qquad \textbf{(D) } \frac{3}{8} \qquad \textbf{(E) } \frac{31}{40}$

Solution

The value, by definition, is \[\frac{3}{8}-\left(-\frac{2}{5}\right)\lfloor{\frac{3}{8}*\frac{-5}{2}\rfloor=-\frac{15}{16}}=\frac{3}{8}-\frac{2}{5}=\boxed{\textbf{(B) } -\frac{1}{40}.}\]